You are on page 1of 113

www.natures.

ir
More Free USMLE , MCCEE ,MCQe and AMQ Flashcards

decreases filling time and decreases


What does tachycardia do to coronary diastole
artery blood flow?
(NOTE: coronary vessels fill in diastole)

LAD
What is the most common site of
(NOTE: ant portion of left ventricle and
coronary artery thrombosis?
ant two thirds of the interventricular
septum)

What is the most common angina pectoris, which is the most


manifestation of coronary artery common type of ischemic heart
disease? disease

age
What is the most important risk factor
(more common in men and peaks in
for angina pectoris?
after age of 60 and peaks after 70 in
women)
stable angina (exercise induced
substernal chest pain)
What is the most common type of
angina? (within one year of stable angina
diagnosis, 20% develop MI or unstable
angina)

Subedocardial ischemia due to


What is the pathogenesis of chronic
decreased coronary artery blood flow
stable angina and what do you see on
or thick muscle wall with ST segment
EKG?
depression
vasospasm at rest with or without
What is the pathogenesis of coronary artery atherosclerosis with
prinzmetal's angina? transmural ischemia and ST segment
elevation

angina at rest; multivessel disease;


disrupted plaques
What are some characteristics of
unstable angina?
(patient will have frequent bouts of
angina at rest with min exertion)

What is a treatment for prinzmetal's calcium channel blockers that


angina? vasodilate coronary arteries

What is seen in patients with chronic replacement of myocardial tissue with


ischemic heart disease? noncontractile scar tissue

Is coronary artery thrombosis usually no, typically cause of death is


present in Sudden cardiac death? ventricular fibrillation

unexpected death within 1 hour after


What is the definition of sudden cardiac the onset of symptoms
death?
(diagnosis of exclusion)

Acute MI
What is the most common cause of
death in the US?
(prominent in males 40-65)
rupture of disrupted plaque -> platelet
thrombus -> AMI
What is the typical pathogenesis of
AMI?
(thomboxane A2 causes vasospasm of
artery to reduce blood flow)

Can cocaine use with normal coronary


Yes
arteries cause AMI?

Q wave = transmural infarction


What type of MI involves Q wave
formation? non Q wave? non-Q wave = subendocardial
infarction

What can increase the short and long Reperfusion injury following
term survival of AMI? thrombolytic therapy

What is contraction band necrosis hypercontraction myofibrils due to


following reperfusion mean? Ca2+
24 hrs - no gross changes but there is
coagulation necrosis

1-3 days - neutrophils abundant and


lyse dead myocardial cells

What are some gross and microscopic 3-7 days - heart is softest and danger
findings of AMI within 24 hours? 1-3 of rupture
days? 3-7 days? 7-10 days? 2 months?
7-10 days - necrotic area yellow and
collagen formation present

2 months - infarcted tissue replace with


white, patchy, noncontractile scar
tissue

What are classic clinical findings in retrosternal pain, radiation to left arm/
AMI? shoulder, diaphoresis

Q wave
Is there a higher mortality rate in Q
wave or non-Q wave AMI? Non Q wave is increased risk for
sudden cardiac death

What is the most common cause of


ventricular fibrillation
death in AMI?

What is the most common time for the


3- 7 days
heart to rupture post AMI?
What are the consequences of a
RCA thrombosis and mitral
posteromedial papillary muscle
regurgitation and LHF
rupture?

embolization because its a thrombus of


What is a danger of a mural thrombus?
a large vessel

Early acute inflammation AMI due to


What type of fibrinous pericarditis increased vessel permeability in
develops 1-7 days of a Q wave AMI? pericardium and exudate of acute
inflammation

6-8 weeks after an AMI and


When does autoimmune pericarditis
autoantibodies direct against damaged
develop?
pericardial antigens (late complication)

CHF occurs due to the lack of


When a ventricular aneurysm forms contractile tissue
following AMI, what is the most
common cause of death? (Rupture is uncommon because of the
good tensile strength of scar tissue)

RHF, hypotension, preserved LV


What are some clinical findings in a RV function
AMI?
(assoc with RCA thrombosis)

creatine kinase isoenzyme (CKMB)


within 4-8 hrs, peaks at 24 hrs

What are the common cardiac lab tests cardiac troponins I and T within 3-12
to run for AMI? hrs, peaks at 24 hrs

LDH 1-2 "flip" within 10 hrs, peaks at 2-


3 days
cardiac troponins I and T
What is the gold standard for diagnosis
(NOTE: these cannot diagnose
of AMI?
reinfarction so CKMB in used in
conjunction with troponins to diagnose)

What do you see in labs if reinfarcation


reappearance of CKMB after 3 days
occurs?

What are classic EKG findings with inverted T waves, elevated ST


AMI? segment, Q waves

chorionic villus
What is the primary site for O2
exchange? (the chorionic villus vessels become
the umbilical vein)

In fetal circulation, what has the highest


umbilical vein
PO2 content?

What are some unique features of fetal foramen ovale and ductus arteriosus
circulation? are patent

What is there an increased risk for with


congential abnormalities
only a single umbilical artery?
What does the ductus arteriosus
become after closure within 2-8 weeks ligamentum arteriosum
post birth?

What is the most common heart congenital heart disease and it


disease in children? increases with increase maternal age

What is a danger of left to right shunts


reversal of shunt and cyanosis
if reversal is not corrected?

ventricular septal defect


What is the most common congenital
heart disease in children?
(defect in the membranous septum)

Atrial septal defect and the most


What is the most common congenital
common type is the patent foramen
heart disease in adults?
ovale (secundum type)

Why is there a fixed splitting of S2 in excess blood in RA causes delay in


ASD? closure of pulmonary valve

What is Patent ductus arteriosus


indomethacin
closed with?
What is differential cyanosis mean and PDA and it means that you have a pink
what is it seen in? upper body and cyanotic lower body

What is the most common cyanotic


tetralolgy of fallot
congenital heart disease?

minimal pulmonary valve stenosis is


absence of cyanosis
What is the importance with the degree
of pulmonary valve stenosis? severe PV stensosi leads to cyanosis
and increased right to left shunting
through VSD

What are the cardioprotective shunts


ASD and PDA
with tetraolgy of fallot?

hypoxic spells caused by sudden


increase in hypoxemia and cyanosis
What are Tet spells?
(squatting increases systemic vascular
resistance, causing temporary reversal
of the shunt)

What are the heart's anatomical


aorta empties RV, pulmonary artery
features in complete transposition of
empties LV, and atria are NORMAL
the great vessels?

Turner's syndrome

(constriction between subclavian artery


What is infantile coarctation associated
and ductus arteriosus in infantile)
with?
(constriction of aorta distal to
ligamentum arteriosum in adult type)
disparity between upper/lower
extremity blood pressure > 10 mmHg
What is clinically seen in adult
coarctation?
also, see a dilation of aorta and aortic
valve ring

decreased blood flow to lower body


Why is hypertension seen in adult cause a decrease renal blood flow and
coarctation? this activated the RAA system causing
hypertension

Anterior intercostal arteries with post


intercostal arteries
What are the types of collateral
circulations seen in adult coarctation?
sup epigastric artery and inf epigastric
artery to external iliac artery

When does acute Rheumatic fever after group A streptococcal pharyngitis,


occur? usually 20 days

immune mediated type II


hypersensitivity reaction because
antibodies cross react with similar
What is the pathogenesis of acute
proteins in human tissue
Rheumatic fever?
also may be cell-mediated immunity
type IV

migratory polyarthritis
What is the most common initial
presentation with RF?
(occurs in large joints, ankles, wrists

myocarditis
What is the most common cause of
death in RF?
(Aschoff bodies present)
In RF, what valves are most often
mitral valve followed by aortic valve
involved?

acute - mitral regurg


What is seen in acute and chronic
attacks of RF?
chronic - mitral stenosis

What are some of the major criteria carditis, arthritis, chorea, erythema
according to the Jones criteria for RF? marginatum, subcutaneous nodules

increased ASO and DNase B titers


What is seen in lab test with RF?
(peak at 4-5 weeks after pharyngitis)

What is the most common cause of


recurrent RF
mitral valve stenosis?

What is a common clinical finding in


atrial fibrillation
mitral stenosis?

Describe the murmur with mitral opening snap followed by an early to


stenosis. mid-diastolic rumble
What is the most common cause of
mitral valve prolapse
mitral regurg?

pansystolic murmur; S3/S4 ; no


Describe the murmur with mitral
increase intensity with deep held
regurg.
inspiration

Marfan and Ehlers Danlos syndromes


What is assoc with mitral valve
prolapse?
(autosomal dominant in some cases)

myxomatous degeneration of mitral


What is the pathophysiology for MVP? valve leaflets and excess production of
dermatan sulfate

Describe the murmur with MVP systolic click followed by murmur

decreased preload causes the click


and murmur to move closer to S1
What do the increased and decreased
preloads assoc with MVP cause?
increased preload causes click and
murmur to move closer to S2

Beta-blocker treatment
What do you do in a symptomatic MVP
(decreases HR and force of contraction
patient?
leading to less stretch and trauma to
the prolapsed leaflets)
What is the most common cause of
aortic valve stenosis in patients > 60 calcific AV stenosis
years old?

What is the most common valvular


lesion causing syncope and angina AV stenosis
with exercise?

What are some clinical findings with microangiopathic hemolytic anemia


aortic stenosis? with schistocytes, hemoglobinuria

isolated aortic valve root dilation


What is the most common cause of
(most common infectious disease
aortic regurg?
cause of aortic regurg is infective
endocarditis)

What does aortic regurg do to the pulse


increases it, produces hyperdynamic
pressure (difference between systolic
circulation
and diastolic pressures)?

early diastolic murmur; bounding


What are some clinical findings with
pulses; S3, S4; no increased intesity
aortic regurg?
with inspiration

What does the presence of an Austin


sign for AV replacement
Flint murmur indicate?
What is the most common cause of stretching of TV ring from RHF,
tricuspid valve regurg? hypertension, dilated cardiomyopathy

Internal jugular Vein (lateral), common


What are 3 structures inside the carotid carotid Artery (medial), vagus Nerve
sheath? (Mnemonic) (posterior)
("VAN")

The acute marginal artery supplies


Right ventricle
which part of the heart?

The circumflex artery (CFX) supplies


Posterior left ventricle
which part of the heart?

The posterior
descending/interventricular artery (PD) Posterior septum
supplies which part of the heart?

The left anterior descending artery Apex and anterior interventricular


(LAD) supplies which part of the heart? septum

What artery supplies the AV and SA


Right coronary artery (RCA)
nodes?
80% of the time it stems from the RCA
Where does the posterior descending (=right dominant). 20% of the time it
(PD) artery arrise from? stems from the circumflex artery (= left
dominant)

Left anterior descending artery (LAD),


Coronary artery occlusion most
which supplies the anterior
commonly occurs where?
interventricular septum

Do coronary arteries fill on systole or


Diastole
diastole?

What is the most posterior part of the The most posterior part is the left
heart? What can enlargement of this atrium. Enlargement can cause
part cause? dysphagia or hoarsness.

CO = (stroke volume)x(heart rate)


What are two ways to calculate cardiac
Fick principle: CO = (rate of O2
output?
consumption)/[(arterial O2 content) -
(venous O2 content)]

MAP = 2/3(diastolic pressure) +


What are 2 ways to calculate mean 1/3(systolic pressure)
arterial pressure (MAP)?
MAP = COx(total peripheral resistance)

Pulse pressure = systolic pressure -


How do you calculate pulse pressure? diastolic pressure. It is proportational to
stroke volume.
What are two ways to calculate stroke
SV = CO/HR = EDV - ESV
volume (SV)?

Stroke Volume is affected by


Contractility, Afterload, and Preload
What are 3 factors that affect stroke "SV CAP")
volume? (Mnemonic) SV increases when preload increases,
afterload decreases, or contractility
increases

What is preload? ventricular EDV

mean arterial pressure (proportional to


What is afterload?
peripheral resistance)

Venodilators (e.g. nitroglycerin)


What is the difference between the
decrease preload;
effects of venodilators and vasodilators
Vasodilators (e.g. hydralazine)
on the heart?
decrease afterload

describes that the force of contraction


What relationship does the Starling
is proportional to the initial length of
curve describe?
cardiac muscle fibers (preload)

EF = SV/EDV = (EDV-ESV)/EDV
How do you calculate ejection faction
(EF)? EF is normally greater than or equal to
55%
delta P = QxR
What equation relates resistance,
delta P = change in pressure
pressure, and flow?
Q = flow
R=resistance

What's a way to measure resistance in Resistance =


a tube using viscosity? What does (8*viscosity*length)/(Pi*r^4)
viscosity mostly depend on in blood
vessels? viscosity depends on hematocrit

What type of blood vessels account for


Arterioles
most of the peripheral resistance?

Which phase of the left ventricle Isovolumetric contraction - period


cardiac cycle has the highest O2 between mitral valve closure and aortic
consumption? valve opening

Mitral and triscupid valve closure.


What makes up the S1 sound?
Loudest at mitral area.

Aortic and pulmonary valve closure.


What makes up the S2 sound?
Loudest at left sternal border.

In early diastole rapid ventricular filling


phase. Associated with increased filling
What makes up the S3 sound?
pressures and more common in dilated
ventricles (but normal in children)
"Atrial kick" - high atrial pressure.
What makes up the S4 sound? Associated with ventricular
hypertrophy.

a wave = Atrial contraction


c wave = RV contraction (tricuspid
What are the a, c, and v waves in the bulging into atrium)
jugular venous pulse? v wave = increased atrial pressured
due to filling against closed tricuspid
valve

Expiration: S1 stays the same, A2 and


How do the physiologic heart sounds P2 (in S2) get closer together
change with inspiration and expiration? Inspiration: S1 stays the same, A2 and
P2 (in S2) get farther apart

Wide splitting, where A2 and P2 are


not as close together as is normal
during expiration and farther apart than Pulmonic stenosis
is normal during inspiration is
associated with what?

Fixed splitting, where A2 and P2 are


always a fixed distance apart
Atrial septal defects (ASD)
regardless of exhalation or inhalation,
is associated with what?

Paradoxical splitting, where P2 comes


before A2, and where the gap between
Aortic stenosis
them is greater on exhalation than
inhalation, is associated with what>

When do you get "Pulsus parvus et Pulses weak compared to heart sounds
tardus"? - aortic stenosis
Mitral/tricuspid regurgitation. Mitral -
What murmur is described as
loudest at apex and radiates toward
"Holosystolic, high-pitched blowing
axilla.
murmur"? Where is it loudest and
Tricuspid - Loudest at tricuspid area
where does it radiate to?
and radiates to right sternal border.

What murmur is described as having a


Aortic stenosis. LV >> aortic pressure
"crescendo-descrescendosystolic
during systole. Has "Pulsus parvus et
ejection murmur following an ejection
tardus" - pulses weak compared to
click (EC) that radiates to
heart sounds.
carotids/apex"?

What murmur is described as a


"holosystolic, harsh sounding murmur VSD
that is loudest at the tricuspid area"?

Mitral prolapse. Enhanced by vasalva


maneuver and decreased by squatting
What murmur is described as a "late
(standing and vasalva decreases the
systolic murmur with midsystolic click
volume of the left ventricle causign
(MC). Loudest at S2" ?
prolapse to occur sooner and more
severely)

What murmur is described as a


Aortic regurgitation. Wide pulse
"immediate high-pitched blowing
pressure when chronic
diastolic murmur"?

What murmur "Follows opening snap Mitral stenosis. Tricuspid stenosis gets
(OS). Delayed rumbling late diastolic louder with inspiration (more blood
murmur"? flows into RA upon inspiration)

What murmur is described as a


"continuous machine-like murmur. PDA
Loudest at time of S2"?
What happens during Phase 0 of the Rapid upstroke - voltage gated Na
ventricular action potential? channels open

Initial repolarization - inactivation of


What happens during Phase 1 of the
voltage-gated Na channels. Voltage
ventricular AP?
gated K channels begin to open.

Plateau - Ca influx through voltage-


gated Ca channels balances K efflux.
What happens during Phase 2 of the
Ca influx triggers Ca release from
ventricular AP?
sarcoplasmic reticulum and causes
myocyte contraction.

Rapid repolarization - massive K efflux


What happens during Phase 3 of the due to opening of voltage-gated slow K
ventricular AP? channels and closure of voltage-gated
Ca channels.

What happens during Phase 4 of the Resting potential - normal K and Na


ventricular AP? permeability

There's only Phase 0, 3, and 4


Phase 0: Upstroke - opening of voltage
gated Ca channels.
Phase 3 = Rapid repolarization (similar
to ventricular AP) Inactivation of the Ca
What phases do pacemaker APs have,
channels and increased activation of
and how are they different than
the K channels --> K efflux
ventricular APs?
Phase 4 = slow diastolic depolarization
- membrane potential spontaneously
depolarizes as Na conductance
increases. Accounts for the
automaticity of SA and AV nodes.
On an ECG, what does the P wave
Atrial depolarization
indicate?

On an ECG, what does the PR interval Conduction delay through the AV node
indicate? (normally < 200 msec)

On an ECG, what does the QRS Ventricular depolarization (normally <


complex indicate? 120 msec)

On an ECG, what does the QT interval Mechanical contraction of the the


indicate? ventricles

On an ECG, what does the T wave


Ventricular repolarization
indicate?

Where is atrial repolarization on the


Masked by the QRS complex
ECG?

On an ECG, what does the ST


Isoelectric, ventricles depolarized
segment indicate?
On an ECG, what is a U wave? Caused by hypokalemia or bradycardia

Associate ventricular tachycardia with Torsades des pointes. Anything that


shifting sinusoidal waveforms on ECG prolongs the QT interval can
that can progress to V-fib with... predispose to torsades des pointes.

When there is an accessory conduction


pathway from the atria to ventricle
(bundle of Kent) that bypasses the AV Creates a delta wave. May result in
node, the ventricles may begin to reentry current leading to
partially depolarize earlier. What supraventricular tachycardia.
characteristic wave is produced, and
what can it cause?

Atrial fibrillation - chaotic and erratic


What does the top line of the ECG
baseline with no discrete P waves in
show? (the bottom line shows sinus
between irregularly spaced QRS
rhythm)
complexes

Atrial flutter - A rapid succession of


identifical, back to back atrial
depolarization waves. The identical
What does this ECG show?
appearnace accounts for the
"sawtooth" appearance of the flutter
waves.

What finding is this? (The bottom line is 1st degree AV block. The PR interval is
clearer) prolonged (>200 msc). Asymptomatic.
2nd degree AV block Mobitz I
(Wenckebach). Progressive
lengthening of PR interval until a beat
What finding does this ECG show?
is "dropped" (a P wave not followed by
a QRS complex). Usually
asymptomatic.

2nd degree AV block Mobitz II


Dropped beats that are not preceded
by a change in the length of the PR
interval (as in type I). These abrupt,
What abnormality does this ECG nonconducted P waves result in a
show? pathologic condition. It is often found
as 2:1 block where there are 2 P waves
to 1 QRS response (the ratio of P:QRS
is always constant). May progress to
3rd degree block.

3rd degree (complete) AV block. The


atria and ventricles beat independently
of each other. Both P waves and QRS
complexes are present, although the P
What condition does this ECG show?
waves bear no relation to the QRS
complexes. The atrial rate is faster than
the ventricular rate. Usually treat with a
pacemaker.

Ventricular fibrillation. A completely


erratic rhythm with no identifiable
What does this ECG show?
waves. Fatl arrhythmia without
immediate CPR and defibrillation.

The baro and chemoreceptors on this


structure transmit via the vagus nerve Aortic arch.
to the medulla.
The aortic arch only respods to
How does BP affect the aortic arch?
increased BP

The baro and chemoreceptors on this


structure transmit via the Carotid sinus
glossopharyngeal nerve to the medulla.

How does BP affect the carotid The carotid sinuses respond to both
sinuses? increased and decreased BP

Decreased BP --> decreased afferent


baroreceptor firing --> increased
efferent sympathetic firing and
What is the effect of hypotension on
decreased efferent parasympathetic
baroreceptors?
stimulation --> vasoconstriction,
increased HR, increased contractility,
increased BP

Respond to decreased PO2 (<


What do peripheral chemoreceptors
600mmHg), increased PCo2, and
respond to?
decreased blood pH

Changes in pH and PCO2 of the brain


What do central chemoreceptors interstitial fluid, which in turn are
respond to? infleunced by arterial CO2. Do not
directly respond to PO2.
Cushing reaction: increased ICP
constricts arterioles --> cerebral
ischemia --> hypertension (sympathetic
What is the Cushing reaction? What response) and reflex bradycardia.
chemoreceptors are responsible for it? (Cushing triad = hypertension,
bradycardia, respiratory depression).
The central chemoreceptors are
responsible.

What organ takes the largest share of


The liver
systemic cardiac output?

Which organ has the highest blood flow


Kidney
per gram of tissue?

Which organ has the highest


The heart
arteriovenous O2 difference?

Pulmonary capillary wedge pressure is Good approximation of left atrial


a good approximation of what? How do pressure. Measured with a Swan-Ganz
you measure it? catheter.

In pulmonary vasculature, hypoxia In pulmonary vasculature, hypoxia


causes what response? In all other causes vasoconstriction. In all other
organs? organs hypoxia causes vasodilation.

What type of shunts cause "blue


Right to left shunts
babies," right to left or left to right?
"The 5 T's"
Tetralogy of Fallot
Transposition of great vessels
What are 5 congenital syndromes with
Truncus arteriosus
right to left shunts? (Mnemonic)
Tricuspid atresia
Total anomalous pulmonary venous
return (TAPVR)

What ist the most common cause of


Tetralogy of fallot
early cyanosis in infants?

In right to left shunts, what may


children do to increase systemic Squat
vascular resistance?

What are causes of 3 left to right


shunts that can lead to "blue kids" (late Frequency - VSD>ASD>PDA
cyanosis)?

People who have initial left to right


shunts will increase pulmonary
vascular resistance due to arteriolar
What is Eisenmenger's syndrome? thickening. Eventually the progressive
pulmonary hypertension will cause the
left to right shunt to become a right to
left shunt.

What is the most common congenital


VSD
cardiac anomal?

What drug can you give to close a


Indomethacin
PDA?
Pulmonary stenosis (most important
determinant for prognosis), Right
What 4 conditions are involved in the ventricular hypertrophy (RVH),
Tetralogy of Fallot? (Mnemonic) Overriding aorta (overrides teh VSD),
VSD
("PROVe")

What is the shape of the heart on an x-


ray in tetralogy of Fallot?
Shaped like a boot

Tetralogy of Fallot is caused by


How does tetralogy of Fallot develop? anterosuperior displacement of the
infundibular septum

The aorta leaves the RV (anterior) and


the pulmonary trunk leaves the LV
What is transposition of the great (posterior). Not compatible with life
vessels? unless a shunt is present to allow
adequate mixing of blood (e.g. VSD,
PDA, or patent foramen ovale).

How does transposition of the great Due to failure of the aorticopulmonary


vessels develop? septum to spiral.

Aortic stenosis proximal to insertion of


What is the infantile type of coarctation
the ductus arteriosus (preductal)
of the aorta? (Mnemonic)
("Infantile: IN close to the heart")

Aortic stenosis is distal to ductus


arteriosus (postductal). Associated with
notching of teh ribs (due to collateral
What is the adult type of coarctation o
circulation), hypertension in the upper
the aorta?
extremities, weak pulses in lower
extremities. Associated with Turner's
syndrome.
Is coarctation of the aorta more
Male to female ratio 3:1
prevalent in males or females?

What drug can be used to cloes a


Close: Indomethacin
PDA? What drug can be used to keep
Keep open: PGE
it open?

22q11 deletion syndromes are (persistent) trucus arteriosus, tetralogy


associated with what heart defects? of Fallot

Down syndrome is associated with ASD, VSD, AV septal defect


what heart defects? (endocardial cushion defect)

Congenital rubella is associated with Septal defects, PDA, pulmonary artery


which cardiac defects? stenosis

Turner's syndrome is associated with


Coarctation of the aorta
which cardiac defect?

Marfan's syndrome is associated with Aortic insufficiency/regurgitation (late


which cardiact defect? complication)
What cardiac defect is associated with
Transposition of great vessels
being born to a diabetic mother?

What is the definition of hypertension? BP > 140/90

What are atheromas? Plaques in blood vessel walls

Atheromas, xanthomas, and corneal


Hyperlipidemia
arcus are all signs of what?

Calcification in the media of the


arteries, especially the radial or ulnar
What is Monckeberg arterioslclerosis?
afteries. Usually benign; "Pipestem"
arteries

What is the name for hyaline thickening


of small arterioles in essential Arteriolosclerosis
hypertension and diabetes mellitus?

Associate hyperplastic "onion skinning"


Malignant hypertension
of arterioles with...
Associate fibrous plaques and
atheromas forming in intima of arteries Atherosclerosis
with...

Associate tearing chest pain radiating


Aortic dissection
to the back with...

How does aortic dissection appear on


Mediastinal widening
x-ray?

What are the 4 most common sites of Abdominal aorta > Coronary artery >
atherosclerosis? Popliteal artery > Carotid artery

What percent of a coronary artery must


be occluded to produce angina > 75%
symptoms?

Associate coronary artery spasm with... Prinzmetal's angina

Death from cardiac causes within 1


What is the definition of sudden cardiac
hour of onset of symptoms, most
death?
commonly due to a lethal arrhythmia
This type of infarct occurs in loose
tissues with collaterals, such as the Red (hemorrhagic) infarcts
liver, lungs, and intestine

This type of infarct occurs in solid


tissues with single blood supply, such Pale
as heart, kidney, and spleen.

An infarct that is reperfused is this color Red ("REd = REperfusion")

What are the 3 most commonly


LAD > RCA > Circumflex
blocked coronary artery branches?

How long after a MI do you first start to


Within 4 hours
see contraction bands?

When do macrophages first start to


By day 5-10
replace neutrophils in a MI?

In the first 2-4 days after a MI, what is


Arrhythmia
there a risk for?
In days 5-10 after a MI, what is there a
Free wall rupture
risk for?

Several weeks after a MI, what is there


Ventricular aneurysm
a risk for?

What is the gold standard for detecting


ECG
a MI within the first 6 hours?

Cardiac troponin (most specific protein


How long after a MI does troponin marker) rises after 3 to 6 hours, peaks
elevate and for how long? at 24 hours, and disappears after 7-10
days

Appears within 4 to 8 hours, peaks at


How long does it take for CK-MB to rise
24 hours,a nd disappears within 1.5 - 3
after a MI?
days

A transmural infarct will have what


ST elevation, pathological Q wave
appearance on a ECG?

A subendocardial infarct will have what


ST depression
appearance on a ECG?
Autoimmune phenomenon resulting in
What is Dressler's syndrome? fibrinous pericarditis (several weeks
post-MI)

What type of cardiomyopathy causes


Dilated (congestive) cardiomyopathy
systolic dysfunction?

What type of cardiomyopathy causes Hypertrophic and restrictive/obliterative


diastolic dysfunction? cardiomyopathies

("ABCCCD")
What are 6 causes of dilated Alcohol abuse, Beriberi, Coxsackie B
cardiomyopathy? (Mnemonic) virus myocarditis, Cocaine use,
Chaga's disease, Doxorubicin

Associate sudden death in young


Hypertrophic cardiomyopathy
athletes with...

beta blockers, or non-dihydropyridine


How do you treat hypertrophic
calcium channel blockers (e.g.
cardiomyopathy?
verapamil)

What does hypertrophic


cardiomyopathy sound like on Loud S4, systolic murmur
auscultation?
Heart failure. They are hemosiderin
laden macrophages (blood backs up
The brown cells in this lung tissue are a into the lungs due to left ventricle
sign of what? failure and macrophages phagocytose
them), and are known as "heart failure
cells"

What is the appearance of the liver in


CHF?
Nutmeg liver

Right heart failure is most often due to


Left heart failure
what?

Isolated right heart failure is usually


Cor pulmonale
due to what?

Fat, Air, Thrombus, Bacteria, Amniotic


What are 6 types of emboli?
fluid, Tumor
(Mnemonic)
("an embolus moves like a FAT BAT")

Fat emboli are associated with what? Long bone fractures and liposuction

DIC can result from what type of


Amniotic fluid
emboli?
Stasis, hypercoagulability, endothelial
What is Virchow's triad, and what can it
damage. Can lead to deep vein
predispose to?
thromboses --> pulmonary embolism.

What is a Roth's spot?


A round white spot on the retina
surrounded by hemorrhage - an
indication of bacterial endocarditis

What are Janeway lesions?


Small erythematous lesions on palm or
sole. A sign of bacterial endocarditis.

What is a splinter hemorrhage?


Hemorrhage on nailbed. A sign of
bacterial endocarditis.

Fever, Roth's spots, Osler's nodes,


What are 8 signs of bacterial Murmur, Janeway lesions, Anemia,
endocarditis? (Mnemonic) Nail-bed hemorrhage, Emboli
("bacteria FROM JANE")

Associate large vegetations on


previously normal valve and rapid S. aureus
onset with...

Associate smaller vegetations on


congenitally abnormal or diseased Strep viridans (and dental procedures)
valves and an insidious onset with..
What is the most common valve
Mitral
involved in bacterial endocarditis?

What is tricuspid valve endocarditis Tricuspid - IV drug abuse ("Don't TRI


associated with? drugs")

What is a disease where you get


Libman-Sacks endocarditis (non SLE ("SLE causes LSE")
bacterial endocarditis)? (Mnemonic)

It is an Aschoff body, and it is


associated with rheumatic endocarditis.
Aschoff bodies are granulomatous
structures consisting of fibrinoid
change, lymphocytic infiltration,
What is this lesion called and what is it
occasional plasma cells, and
an indication of?
characteristically abnormal
macrophages surrounding necrotic
centres. Some of these macrophages
may fuse to form multinucleated giant
cells.

They are Anitschkow cells, and they


are pathognomonic for rheumatic
endocarditis. The cells are also called
What type of cells are these, and what
caterpillar cells, as they have a large
do they indicate?
amount of clear cytoplasm surrounding
a rod-shaped nucleus that to some
resembles a caterpillar.
Fever, Erythema marginatum, Valvular
damage, ESR increase, Red-hot joints
What are 7 symptoms of rheumatic (polyarthritis), Subcutaneous nodules
fever? (Mnemonic) (Aschoff bodies), St. Vitus' dance
(chorea)
("FEVERSS")
Associate electrical alternans (beat-to-
beat alternations of QRS complex
height), pulsus paradoxus
Cardiac tamponade
(exaggerated decrease in amplitude of
pulse durin inspiration), and distant
heart sounds with...

What type of pericarditis is caused by


SLE, rheumatoid arthritis, viral Serous pericarditis
infection, and uremia?

What type of pericarditis is caused by


Fibrinous pericarditis
uremia, MI, and rheumatic fever?

What type of pericarditis can be caused


Hemorrhagic pericarditis
by TB and malignancies?

Tertiary syphilis disrupts the vasa


vasorum leading to dilation of the aorta
Syphilis causes dilation of the aorta
and valve ring. Can result in aneurysm
and valve ring via what mechanism?
of the ascending aorta or aortic arch
and aortic valve incompetence.

What is the most common primary


cardiac tumor in adults?
Myxomas. They are a a benign tumor
usually described as a "ball valve"
obstruction in the atria (mostly LA).
What is the most common primary Rhabdomyomas - associated with
cardiac tumor in children? tuberous sclerosis.

What are the most common heart


Metastases (melanoma, lymphoma)
tumors overall?

Associate arteriovenous malformation


in small vessels that look like dilated Telangiectasia. Affects small vessels.
capillaries with

Associate decreased blood flow to the


skin due to arteriolar vasospasm in Raynaud's disease. Affects small
response to cold temperature or vessels.
emotional stress with...

Associate focal necrotizing vasculitis,


necrotizing granulomas in the lung and
Wegner's granulomatosis
upper airway, and necrotizing
glomerulonephritis with...

What is a strong marker for Wegner's


c-ANCA
granulomatosis?

How do you treat Wegner's


Cyclophosphamide and corticosteroids
granulomatosis?
Microscopic polyangitis, primary pauci
What are 3 pANCA positive immune crescentic glomerulonephritis
vasculitides? (pauci immune = paucity of antibodies),
Churg-Strauss syndrome.

Associate granulomatous vasculitis


with eosinophilia, invovling lung, heart,
Churg-Strauss syndrome
skin, kidneys, and nerves, and often
seen in atopic patients with...

Sturge-Weber disease
Associate port-wine stain on face and
leptomeningeal angiomatosis
(intracerebral AVM) with...

Henoch-Schonlein purpura. Common


triad: skin, joints, GI
Associate palpable purpura after a URI
with...

What is the most common form of


Henoch-Schonlein purpura
childhood systemic vasculitis?

Associate idiopathic, segmental,


Beurger's disease (thromboangiitis
thrombosing vasculitis of small and
obliterans) - affects small and medium
medium peripheral arteries and veins,
sized vessels
often seen in heavy smokers with...

Intermitten claudication, superficial


What are 3 symptoms of Buerger's
nodular phlebitis, cold sensitivity
disease?
(Raynaud's phenomenon)
What is the treatment?
Treatment: Smoking cessation
Associate acute, self-limiting
necrotizing vasculitis in infants and kids
that presents with fever, congested Kawasaki disease
conjunctiva, changes in lips/oral
mucosa ("strawberry tongue") with...

Children with Kawasaki disease are at


Coronary aneurysms
risk for what?

What size arteries does Kawasaki


Small and medium vessels
disease affect?

Associate necrotizing immune complex


inflammation of medium-sized
muscular arteries associated with Polyarteritis nodosa
Hepatitis B seropositivity in 30% of
patients with...

How do the ages of the lesions appear Lesions are of different ages.
in Polyarteritis nodosa? Typically involves renal and visceral
What vessels are typically affected? vessels

How do you treat polyarteritis nodosa? Corticosteroids, cyclophosphamide

Associate granulomatous thickening of


aortic arch and/or proximal great Takayasu's arteritis
vessels with...
Fever, Arthritis, Night sweats, MYalgia,
What are 7 symptoms of Takayasu's SKIN nodules, Ocular disturbances,
arteritis? (Mnemonic) Weak pulses in upper extremities
("FAN MY SKIN On Wednesday")

What's the most common vasculitis


that affects medium and large arteries, Temporal arteritis (giant cell arteritis)
usually branches of the carotid artery?

Unilateral headache, jaw claudication,


imparied vision (occlusion of opthalmic
What are common symptoms of
artery)
temporal arteritis (giant cell arteritis)?
("TEMporal arteritis has signs near
TEMples")

How do you treat temporal arteritis? High-dose steroids

The volume of blood in the arteries


What is stressed volume?
(because it is under high pressure)

What adrenergic receptors are found in


alpha-1 adrenergic receptors: causes
arterioles of the skin, splanchnic, and
constriction
renal circulations?

What adrenergic receptors are found Beta-2 adrenergic receptors: causes


on the arterioles of skeletal muscle? relaxation
The highest proportion of blood in the
CV system is found in what type of Veins (called "unstressed volume")
vessels?

v = Q/A
where:
What is an equation for blood flow
v = velocity (cm/sec)
velocity in the blood vessles?
Q = blood flow (mL/min)
A = cross sectional area (cm^2)
Illustrated by systemic circulation:
1/Rtotal = 1/Ra + 1/Rb + ... 1/Rn
What is the formula for parallel where:
resistance? Ra, Rb, and Rn are the resistances of
the renal, hepatic, and other arteries
respectively

Illustrated by arrangement of blood


What is the formula for series vessels within a given organ:
resistance? Rtotal = Rartery + Rarterioles +
Rcapillaies

In each parallel artery, ____ is the


Pressure
same

In each series artery, ____ is the same Flow


Reynold's number predicts whether
blood flow will be laminar or turbulent.
When Reynold's number is increased,
there is greater tendency for
turbulence.
What is Reynold's number and what
Reynold's number (and therefore
affects it?
turbulence) is increased by:
Decreased blood viscosity (e.g.
decreased hematocrit, anemi), and
increased blood velocity (e.g.
narrowing of a vessel)

Capacitance describes the


What is capacitance? Capacitance is distensability of blood vessels.
inversely related to what? Capacitance is inversely related to
elastance, or stiffness.

C = V/P
where:
What is the formula for Capacitance? C= capacitance (mL/mm Hg)
V = volume (mL)
P = pressure (mm Hg)

Much greater for veins.


Is capacitance greater for arteries or
Capacitance of blood vessels decrease
veins? How does capacitance change
with age, leading to an increase in
with age?
blood pressure

Aorta, 100 mm Hg
What are the mean pressures in the
Arterioles, 50 mm Hg
aorta, arterioles, capillaries, and vena
Capillaries, 20 mm Hg,
cava?
Vena cava, 4 mm Hg
ARP - Reflects the time during which
no action potential can be initiated,
regardless of how much inward current
is supplied.
What is the difference between
ERP - Is the period during which a
absolute refractory period (ARP),
conducted action potential cannot be
Effective refractory period (ERP), and
elicited.
Relative refractory period (RRP)?
RRP - Is the period during which an
action potential can be elicited, but
more than teh usual inward current is
required.

Chronotropic effects produce changes


What are chronotropic effects and how in heart rate. A positive chronotropic
are they produced? effect increases heart rate by
increasing the firing of the SA node.

Dromotropic effects produce changes


in the conduction velocity, primarily in
What are dromotropic effect? the AV node. (Note: negative
dromotropic effects increase the PR
interval)

SA node, atria, and AV node have


Which parts of the heart have parasympathetic vagal innervation. The
parasympathetic vagal innervation? ventricles do not. The neurotransmitter
Which part doesn't? is ACh which acts at muscarinic
receptors.

What part of the SA node action


Phase 4 depolarization (decreased
potential is altered in producing
heart rate by decreasing the Na influx)
chronotropic effects?

Which part of the AV node action


Decreased inward Ca current and in
potential is affected to produce
phase 0
dromotropic effects?
The presence of gap junctions that act
What accounts for the observation thta
as low-resistance paths between cells
the heart behaves as an electrical
that allow for rapid electrical spread of
syncytium?
APs

What structures in heart sarcomeres


invaginate the cells at the Z lines and T tubules
carry APs into the cell interior?

What structure is the site of storage


and release of Ca for excitation- Sarcoplasmic reticulum
contraction coupling?

How do positive inotropic agents affect


They increase contractility
the heart?

They inhibit the Na,K-ATPase in the


myocardial cell membrane -->
increased intracellular [Na], diminishing
the Na gradient across the cell
How do cardiac glycosides increase
membrane. Na+-Ca+ exchange (the
contractility of the heart?
mechanism that extrudes Ca from the
cell), depends on the size of the Na
gradient and thus is diminished,
producing increased intracellular [Ca]

This is a LV pressure-volume loop. Mitral valve opens at "d."


Where does the mitral valve open and Mitral valve closes at "a."
close? Where does the aortic valve Aortic valve opens at "b."
open and close? Aortic valve closes at "c."
Detects the integrity of the
baroreceptor mechanism. Vasalva
maneuver --> increased intrathoracic
pressure --> decreased venous return -
-> decreased cardiac output and
arterial pressure
What does the vasalva maneuver
If the baroreceptor mechanism is
detect?
inatact, the decrease in arterial
pressure is sensed by the
baroreceptors, leading to an increase
in sympathetic outlfow to the heart and
blood vessels, causing an increase in
heart rate.
Is released from the atria in response
to an increase in blood volume and
atrial pressure:
What are 3 actions of atrial natriuretic causes relaxation of vascular smooth
peptide? muscle
causes increased excretion of Na and
water by the kidney
Inhibits renin secretion

Caues local relaxation of vascular


What does endothelium-derived
smooth muscle. One form of EDRF is
relaxing factor (EDRF) do?
NO.

If perfusion pressure to an organ is


suddenly decreased, compensatory
vasodilation of the arterioles will occur Autoregulation
to maintain a constant flow. This is
called....

If metabolic activity in an organ


increases, blood flow will increase to
Active hyperemia
that organ proportionately to meet
metabolic demands. This is called...
An increase in blood flow to an organ
after a period of occlusion of flow is Reactive hyperemia
called...

What are 2 vasoactive metabolites in


Hypoxia, adenosine
the coronary arteries?

What are 2 vasoactive metabolites in


CO2, H+
the cerebral circulation?

What are 3 vasoactive metabolites in


Lactate, K+, adenosine
skeletal muscle?

What is the most important method of Rest: sympathetic control


controlling blood flow to skeletal Exercise: Local metabolic control
muscle at rest? During exercise? (lactate, K+, adenosine)

Is based on the observation that


vascular smooth muscle contracts
What is the myogenic hypothesis for
when it is stretched. Explains
local control of blood flow?
autoregulation, but not active or
reactive hyperemia.
Tissue supply of O2 is matched to the
tissue demand for O2. Vasodilator
What is the metabolic hypothesis for metabolites, produced when there is
local control of blood flow? metabolic activity, mediates increased
O2 supply during increased O2
demand.
Causes arteriolar dilation and venous
What is the effect of histamine on blood
constriction --> increased capillary
vessels?
pressure --> local edema

Causes arteriolar dilation and venous


What is the effect of bradykinin on constriction --> increased capillary
blood vessels? pressure --> local edema (similar to
histamine)

Causes arteriolar constriction and is


released in response to blood vessel
What is the effect of serotonin on blood
damage to help prevent blood loss.
vessels?
Has been implicated in the vascular
spasms of migraine headaches.
What is the effect of the following
Prostacyclin: Vasodilator
prostaglandins on blood vessels:
E-series prostaglandins: Vasodilator
Prostacyclin
F-series prostaglandins:
E-series prostaglandins
Vasoconstrictor
F-series prostaglandins
Thromboxane A2: Vasoconstrictor
Thromboxane A2

What is coronary circulation controlled Almost exclusively by local metabolic


by? factors (hypoxia, adenosine)

Sympathetic control (cold temp -->


What is the main method of blood
sympathetic activation -->
vessel regulation in skin?
vasoconstriction)

Calculated LDL = cholesterol - HDL -


triglyceride/5
How do you calculate LDL in the
blood? How does the presence of
Chylomicrons falsely lower calculated
chylomicrons affect this calculation?
LDL by increasing diet-derived
triglycerid
What molecule can help remove
cholesterol from atherosclerotic HDL
plaques for disposal in the liver?

What lipoprotein is required for


B-48
chylomicron assembly?

What lipoprotein is required for VLDL


B-100
assembly and secretion?

How does hypertriglyceridemia affect


Causes turbidity
the appearance of plasma?

Diet derived triglycerides are found in... Chylomicrons

Liver-derived triglycerides are found


VLDL
in...

Increased LDL due to decreased LDL


What is Type II hyperlipoproteinemia? receptors
What is an example? (E.g. familial hypercholesterolemia -
Autosomal Dominant)
Familial dysbetaliporpoteinemia
What is type III hyperlipoproteinemia Accumulation of "remnants"
and what is the problem? What's an (chylomicron and IDL)
example due to deficiency of alipoprotein E
(apoE)

Accumulation of VLDL due to


increased in synthesis or decrease in
What is Type IV hyperlipoproteinemia?
catabolism. Type IV is the most
common lipid disorder.

What is the most common cause of


Most common cause is alcohol excess
Type IV hyperlipoproteinemia?

Deficiency of chylomicrons, VLDL, and


LDL (decreased cholesterol and
triglycerides). Leads to malabsorption,
What are signs of apolipoprotein B because chylomicrons accumulate in
deficiency? villi and prevent reabsorption of
micelles. Marked decrease in vitamin
E. Ataxia, hemolytic anemai, and
thorny RBCs (acathocytes).

Endothelial cell injury (e.g. due to


What is the first step in
smoking, hypertension, homocysteine,
atherosclerosis?
LDL)

Fibrous cap (plaque) composed of


What's the pathognomonic lesion of smooth muscle, foam cells,
atherosclerosis? inflammatory cells, and extracellular
matrix

What is an excellent marker of


C-reactive protein
disrupted atherosclerotic plaques?
What is the most common site for
Abdominal aorta
atherosclerosis?

What are two diseases associated with


Diabetes mellitus, hypertension
hyaline arteriolosclerosis?

What's the most common aneurysm in


Abdominal aortic aneurysm
men older than 55 years of age?

How does vessel wall diameter affect Increased diamete causes increased
vessel wall stress? vessel wall stress

What is the triad of a ruptured Left flank pain, hypotension, pulsatile


abdominal aortic aneurysm? mass

Vessel wall weakening due to an


What is a mycotic aneurysm?
infection

At the junction of communicating


Where are berry aneurysms found?
branches with main cerebral vessels
Where does the blood go in a ruptured
Subarachnoid space
berry aneurysm?

Aortic arch aneurysm is associated Syphilis (tertiary) - caused by vasculitis


with which organism? of the vasa vasorum

There is increased stroke volume


because the back flow of blood
How does aortic regurgitation create increases LVEDV. After the heart
such a large pulse pressure? beats, there is a drop in diastolic
pressure due to blood rapidly draining
back into the left ventricle.

What is the most common cause of


death in Marfan syndrome and Ehlers- Aortic dissection
Danlos Syndrome?

Men with a mean age of 40 to 60 years


What is the most common group to with antecedent hypertension (also
have aortic dissections? common during pregnancy due to 2X
higher plasma volume)

Cystic medial degeneration/necrosis.


What is the histologic hallmark change
Elastic tissue fragmentation leads to
in patients with Marfan syndrome who
matrix material collecting in areas of
have aortic dissections?
fragmentation in the tunica media

What is the most common cause of


Cardiac tamponade
death in aortic dissection?
Associate severe retrosternal chest
pain radiating to the back ('tearing
pain") and loss of upper extremity Aortic dissection
pulse (or pulse differences between left
and right arms) with...

What type of valve dysfunction can you Aortic valve regurgitation due to aortic
get in aortic dissection? valve ring dilation

What is the most common site of


Superficial saphenous veins
varicose veins?

Thrombosis of a vein without


What is phlebothrombosis?
inflammation

What is the most common cause of


Stasis of blood flow. Most common in
phlebothrombosis? What is the most
the deep vein of the calf.
common site of phlebothrombosis.

Stasis dermatitis, an orange


discoloration (hemosiderin) around the
ankles caused by rupture of the Deep vein thrombosis
penetrating branches is a sign of what
condition?

Carcinoma of the pancreatic head -


Superficial migratory thromboplebitis is
due to release of thrombogenic
associated with what?
substances by cancer
Superior vena cava syndrome (pic on
the left), which includes puffiness and
Primary lung cancer (90% of cases)
blue to purple discoloration of the face,
usually a small cell carcinoma of the
arms, and shoulders due to extrinsic
lung
compression of the superior vena cava,
is most often caused by what?
Compression of the neurovascular
compartment of the neck that leads to
vascular signs (arms falling asleep
while person is sleeping), nerve root Thoracic outlet syndrome
signs (e.g. numbness, parasthessias)
sometimes due to a cervical rib is
called...

Positive Adson test - pulse disappears


What is a test for thoracic outlet when the arm is outstretched and the
syndrome? patient looks to the side of the
outstretched arm

Lymphangitis, inflammation of the


lympathics that often appears as a "red
streak" (in this picture it is going to the Cellulitis caused by Strep. pyogenes
patient's axilla) is most often caused by
what?

Lymphedema of the hands and feet in


Turner's. Also associated with webbed
newborns is associated with which
neck due to a lymphatic abnormality.
syndrome?

How do you treat capillary No need to. They normally regress with
hemangiomas in newborns? age.
c-ANCA antibodies are against what Proteinase 3 (e.g. found in Wegener's
structure? granulomatosis)

myeloperoxidase (e.g. found in Churg-


What are p-ANCA antibodies against? Strauss syndrome, microsopic
polyangitis)

Fibrosis is usually associated with what


Medium-sized vessel vasculitis
size vessel disease?

Large vessel vasculitis is usually


associated with what histological Granulomas
structures?

Polyarteritis nodosa always spares


Pulmonary arteries
what arteries?

Strawberry tongue and MIs in children


Kawasaki disease
are associated with which vasculitis?

Can you give corticosteroids in No, contraindicated because of danger


Kawasaki disease? of vessel rupture
Raynaud's phenomenon is secondary
What's the difference between to other diseases (e.g. systemic
Raynaud's disease and Raynaud's sclerosis, CREST syndrome) whereas
phenomenon? Raynaud's disease is a primary
disease

Systolic blood pressure correlates


Stroke volume
with...

Diastolic blood pressure correlates tonicity of total peripheral resistance


with... arterioles

Causes them to vasoconstrict. Sodium


enters the arteriole smooth muscle
What is the effect of excess sodium on
cells and opens calcium channels,
TPR arterioles?
causing vasoconstriction. Increases
diastolic blood pressure

Essential hypertension (95% of HTN) -


What is the most common type of
genetic factors reduce renal sodium
hypertension?
excretion

Hypertension due to activation of renin-


angiotensin-aldosterone system. E.g.
Atherosclerotic plaque (in elderly men)
or fibromuscular hyperplasia (in middle
What is renovascular hypertension? aged women) partially blocks blood
flow in the renal artery, leading to
activation of the renin-angiotensin-
aldosterone system in the affected
kidney.
What is the most common cause of Oral contraceptives: increased
hypertension in young women? synthesis of angiotensinogen

What is the most common overall


Left ventricular hypertrophy
complication of hypertension?

What is the most common cause of


Acute MI
death in hypertensive patients?

Causes sarcomere duplication.


How does heart wall stress affect the
Duplication in parallel thickens muscle.
muscle?
Duplication in series lengthens muscle.

What is the most common cause of left


Essential hypertension
ventricular hypertrophy?

Increased afterload causes what type


Concentric hypertrophy
of hypertrophy?

Increased preload causes what type of


Eccentric (dilation)
hypertrophy?
S4 heart sound - correlateds with atrial
What sound is produced in ventricular contraction in late diastole. Caused by
hypertrophy? blood entering a noncompliant
ventricle.

What is the most common type of


Left sided heart failure
CHF?

The heart fails when it is unable to


What is the general cause of heart
eject blood delivered to it by the
failure?
venous system

"Forward failure" is another name for Left-sided heart failure --> pulmonary
what? What does it cause? edema.

Systolic dysfunction is decreased


ventricular contraction (decreased
What is the difference between systolic
ejection fraction). Diastolic dysfunction
and diastolic dysfunction?
is increased resistance to filling of the
ventricle (EF is normal).

S3 heart sound. Occurs in early


diastole. Caused by blood entering a
What is the first cardiac sign of LHF? volume-overloaded left ventricle. The
intensity of the heart sound increases
with expiration.

Expiration increases the intensity of


what sort of heart murmurs and Left sided ones
abnormal heart sounds?
Lying down increases venous return to
the right side of the heart. If there is
What causes paroxysmal nocturnal
LHF, blood backs up into the lungs
dyspnea?
producing pulmonary edema and the
person cannot breathe.

Also known as right sided heart failure.


Backward failure is also known as
-->increase in venous hydrostatic
what? What does it cause?
pressure

Inspiration increases the intensity of


what sorts of murmurs and abnormal Right sided ones
heart sounds?

Form of heart failure in which cardiac


output is increased compared
withvalues for the normal state (e.g.
What is high-output heart failure? due to increased stroke volume,
decreased blood viscosity, vasodilation
of peripheral resistance arterioles, or
arteriovenous fistula)

What is the most common cause of


Trauma from a knife wound
arteriovenous fistula?

What effect does tachycardia have on Decreases filling time, leading to


the coronary arteries? ischemia

What is the most common


manifestation of coronary artery Angina pectoris
disease?
What does stable angina look like on a Shows subendocardial ischemia with
ECG? ST-segment depression

What does Prinzmetal's angina look Transmural ischemia with ST-segment


like on ECG? elevation

Describe the general sequence of Rupture of disrupted plaque --> Platelet


pathology in a MI thrombus --> Acute MI

A Q-wave infarction is a... Transmural infarction

A non-Q wave infarction is a ... Subendocardial infarction

In elderly or in individuals with diabetes


In what population do "silent MI's"
mellitus due to high pain threshold or
occur?
problems with the nervous system

What is the most common cause of


Ventricular fibrillation
death in acute MI?
What is the most common arrhythmias
Ventricular premature contractions
post MI?

What is the most common cause of


CHF due to lack of contractile tissue
death after ventricular aneurysms?

Wall rupture most commonly occurs


3 to 7 days
how long after a MI?

Fibrinous pericarditis with or without


effusion commonly occurs how long 1 to 7 days
after a MI?

When do ventricular aneurysms Clinically recognized within 4 to 8


commonly occur after a MI? weeks

Normally, LDH2 is higher than LDH1.


In acute MI, LDH1 in cardiac muscle is
What is the (LDH)1-2 "flip" that occurs
release causing the "flip". LDH1-2
during an acute MI?
appears within 10 hours, peaks at 2-3
days, and disappers within 7 days
Inverted T waves - correlate with areas
of ischemia at periphery of infarct.
Elevated ST segment - correlates with
How do ECG changes correlate with
injured myocardial cells surrounding
microscopic changes after a MI?
the area of necrosis.
New Q waves - correlate with the area
of coagulation necrosis.
SVC: Right atrium to right ventricle
In fetal circulation, blood that comes
IVC: Right atrium to Left atrium via the
from the SVC and IVC go where?
foramen ovale

Shunted through the patent ductus


In fetal circulation, where does most of
arteriosus into the aorta. Kept open by
the pulmonary artery blood go?
prostaglandin E2.

What vessels in fetal circulation have


Highest: Umbilical vein
the highest and lowest oxygen
Lowest: Umbilical arteries (two of them)
concentrations?

What is the most common congenital


Ventricular septal defect (VSD)
heart disease in children?

What is the most common congenital


Atrial septal defects (ASD)
disease in adults?

Associate a pink upper body and a


patent ductus arteriosis
cyanotic lower body in an infant with...

In Tetralogy of Fallot, what correlates


with the presence or absence of The degree of pulmonary stenosis
cyanosis?
A type of cyanotic congenital heart
What is total anomalous pulmonary disease:
venous return? Pulmonary vein empites oxygenated
blood into the right atrium.

A disparity between upper and lower


extremity blood pressure > 10 mm Hg Adult coarctation of the aorta
is a sign of what?

What type of valve lesions are typical Mitral regurgitation in acute attack,
of Rheumatic fever? mitral stenosis in chronic attack

Opening snap followed by a mid-


Mitral stenosis
diastolic rumble is associated with...

What is the most common cause of


Mitral valve prolapse
mitral regurgitation?

Leads to left ventricular hypertrophy,


because mitral regurgitation leads to
How does mitral regurgitation affect the
increased atrial volume, which leads to
left ventricle?
volume overload of the left ventricle -->
LHF

Mitral valve prolapse is associated with


Marfan and Ehlers-Danlos syndrome
what two diseases?
It is due to myxomatous degneration
What is the pathophysiology of mitral
(pathologic weakness of CT) due to
valve prolapse?
excess production of dermatan sulfate

Mid-systolic click due to sudden


What sound is produced in mitral restrain by the chordae of the
prolapse? prolapsed valve, followed by mid to late
systolic regurgitant murmur

What is the most common cause of Calcification of normal or bicuspid


aortic stenosis? valve

What is the most common valvular Aortic stenosis. Decreased blood flow
lesion causing syncope and angina through the stenotic valve leads to
with exercise? decreased blood flow to the brain.

What is the most common cause of


Isolated aortic valve root dilation.
aortic regurgitation?

What is the most common cause of


Infective endocarditis.
acute aortic regurgitation?

Early diastolic murmur associated with


Aortic regurgitation
bounding pulses is indicative of...
Regurgitant stream from incompetent
What is an Austin Flint murmur and
aortic valve hits the anterior mitral valv
what does it indicate?
leaflet producing a diastolic murmur

Tricuspid valve regurgitation and


pulmonary valve stenosis. Liver
Carcinoid tumor is associated with metastasis from a carcinoid tumor of
what type of valve disease? the small intestine secretes serotonin,
which causes fibrosis of the tricuspid
valve and pulmonary valve.

What is the most common cause of


Strep. viridans
infective endocarditis?

What is the most common cause of


Staph aureus
infective endocarditis in IV drug users?

What is the most common cause of


infective endocarditis due to prosthetic Staph epidermidis
devices?

What is the most common cause of


infective endocarditis in ulcerative Strep bovis
colitis or colorectal cancer?

What valve is most often affected in


Tricuspid
infective endocarditis in IV drug users?
Nonbacterial thrombotic endocarditis,
where circulating mucin from mucin-
producing tumors create sterile, Colon, pancreas
nondestructive mitral valve vegetations
is associated with which cancers?

What is the most common cause of


Coxsackie
myocarditis and pericarditis?

A young woman with pericarditis and


SLE
effusion most likely has...

What is the most common cause


TB
worldwide of constrictive pericarditis?

What is the most common


Dilated
cardiomyopathy?

Mutations in the heavy chain of beta-


What causes the familial form of
myosin and in the troponins. Autosomal
hypertrophic cardiomyopathy?
dominant.

What is the most common cause of


Hypertrophic cardiomyopathy
sudden death in young individuals?
What is the most common part of the
Pericardium
heart for metastasis?

What are the main primary cardiac Adults: Myxomas


tumors in adults and children? Children: Rhabdomyomas

Quinidine, Amiodarone, Procainamide,


What are 4 class IA antiarrhythmics? Disopyramide
(Mnemonic) ("Queen Amy Proclaims Diso's
Pyramid")

What are 3 class IB antiarrhythmics? Lidocaine, Mexiletine, Tocainide


(Mnemonic) "I'd Buy Lidy's Mexican Taco's"

Decreased CO (due to decreased


How are cardiac output (CO), left blood volume)
ventricular end diastolic pressure Decreased LVEDP
(LVEDP) and pulmonary vascular Increased PVR (due to
resistance (PVR) affected in vasoconstriction from catecholamines
hypovolemic shock? and angiotensin II, released due to
decreased CO)
Decreased CO (due to decreased force
of contraction from left ventricle - e.g.
How are cardiac output (CO), left
MI)
ventricular end diastolic pressure
Increased LVEDP (blood accumulates
(LVEDP) and pulmonary vascular
in LV)
resistance (PVR) affected in
Increased PVR(due to vasoconstriction
cardiogenic shock?
from catecholamines and angiotensin
II, released due to decreased CO)
Increased CO (due to rapid blood flow
through dilated arterioles caused by
endotoxins, and anaphylatoxins like
How are cardiac output (CO), left
C3a and C5a)
ventricular end diastolic pressure
Decreased LVEDP (due to neutrophil
(LVEDP) and pulmonary vascular
transmigration through pulmonary
resistance (PVR) affected in septic
capillaries into alveoli producing
shock (initial phase)?
noncardiogenic pulmonary edema)
Decreased PVR (due to vasodilation of
peripheral resistance arterioles)

What's the most common cause of


Multiorgan dysfunction
death in shock?

non-enzymatic glycosylation renders


what is the mechanism of small vessel
vessels permeable to probein --> gives
injury in diabetes?
hyalinization and narrows lumen

shrunken cobblestones appearance


what is gross appearance of kidney due to lacunar strokes and
with HTN? hyalinization of arterioles and
capillaries

onion-skinned appearance of small


malignant hypertension
vessels in kidney

why is the abdominal aorta the most absence of vaso vasorum... has no
common site of aneurysm? blood supply below renal arteries
sudden onset of severe left flank pain,
hypotension, pulsatile mass on PE ruptured abdominal aortic aneurysm
means what?

what is underlying pathology of vasculitis (all of syphilis is vasculitis)


syphilitic lesions? treponeme infects vaso vasora

key factors for causing dissecting aortic


hypertension
aneurism

what is pathogenesis of dissecting elastic tissue fragmentation, cystic


aortic aneurysm medial necrosis (GAGs)

most common type of aortic proximal dissection --> occludes upper


dissection? arteries, causes absent pulse on left

what conditions predispose to aortic marfan syndrome, ehlers danlos


dissection? syndrome, pregnancy

vascular malformation in trigeminal


region of face. also has AV
sturge weber...what is it? malformation in brain on same side.
predisposes to bleeding. people are
retarded
how do you treat capillary
leave it alone
hemangiomas?

cause of kaposi's sarcoma herpes virus 8

lesion only seen in AIDS patients that


vacillary angiomatosis caused by
looks like kaposi's sarcoma but isn't.
bartonella henselae
what is it?

angiosarcoma of liver is caused by


arsenic and polyvinyl chloride
what?

type III hypersensitivity --> immune


small vessel vasculitis is caused by
complex deposition --> palpable
what?
purpura

most common cause of coronary artery


kawasaki's
disease in children

what lesion do you see in muscular


infarction
arteries?
what kind of vessels are affected in
elastic arteries (takayasu's)
people with no pulse?

granulomatous inflammation
temporal arteritis caused by what? (multinucleated giant cells present) of
temporal artery

digital necrosis of fingers and toes in


buerger's disease
heavy smoker

14 y/o boy with URI one week ago


presents with polyarthritis, joint pains,
henoch-schonlein purpura --> IgA Abs
hematuria with RBC casts and palpable
in small vessels
purpura of buttocks and lower leg. what
is the cause?

most common vasculitis in children? henoch-schonlein purpura

saddle nose deformity, chronic sinus


infections, nodular masses in lungs, wegener's granulomatosis --> C-ANCA
glomerular disease. what test will --> treat with cyclophosphamide
diagnose?

polyarteritis nodosa. what antibody is P-ANCA --> associated with HBV


associated with this? surface antigen
IV drug abuser with chronic hepatitis B
has nodular inflamed mass on lower
polyarteritis nodosa
extremity and hematuria. what does he
have?

diabetic ketoacidosis with frontal lobe


cerebral abscess related to what mucormycosis
fungus?

what are the spirocetes? leptospira, borrellia, treponeme

cryoglobulinemia associated with what


HCV
hepatitis?

digital vasculitis leads to fibrosis.


manifests as Raynaud's syndrome. scleroderma
what is is?

Calcinosis/Centromere Ab
Raynaud's
CREST syndrome associated with
Esophageal dysmotility
what?
Sclerodactyly
Telangiectasia

cold-reacting antibodies, vasculitis,


three causes of raynaud's?
vasoconstriction
essential hypertension has what renin
low
level in blood?

where do most hypertensive ruptures


in branches from middle cerebral artery
occur in brain?

concentric hypertrophy is caused by


increased after-load
what?

dilated hypertrophy is caused by what? increased preload (volume overload)

occurs in early diastole, so it must be


what causes and S3 heart sound?
caused by volume overload

occurs in late diastole --> problem with


S4 heart sound is caused by what? compliance (atrium encountering a
problem filling the ventricle)

what abnormal heart sound is found in


S4
hypertension?
all right sided heart murmurs and
abnormal heart sounds increase in inspiration (due to increased filling)
intensity during what?

aortic valve is heard where? right second intercostal space

pulmonic valve heard best where? left second intercostal space

mitral valve heard best where? apex

tricuspid valve is heard best where? right sternal border

left sided heart murmurs and abnormal


expiration (increased intrapleural
heart sounds are increased in intensity
pressure)
during what?

IV drug abuser with fever, pansystolic


murmur along parasternal border that
increases on inspiration, distended infective endocarditis of tricuspid valve
neck veins. what is most likely
diagnosis?
bruit over area, pulsatile, pressing on it
signs of AV fistula
causes decrease in HR

O2 sat on right side is what? 75%

O2 sat on left side of heart is what? 95%

tardive cyanosis. --> reversal of L-to-R


eisenmenger's syndrome is what?
shunt

what is most common teratogen with


Fetal alcohol syndrome
ASD associated?

murmur heard btwn shoulder blades


patent ductus arteriosus
caused by what?

teratogen associated with patent


congenital rubella
ductus
aorta straddles septum, VSD, pulmonic
4 things in tetralogy of Fallot stenosis (below valve), pulmonic
hypertrophy

what in tetralogy of Fallot causes


degree of pulmonic stenosis
cyanosis?

which shunts are protective in tetralogy


ASD and patent ductus
of fallot?

what abnormalities do you get with polycythemia and high risk for infective
right-to-left shunt? endocarditis

Chest pain induced by exercise or


emotions. ST segment depression
Clinical features of stable angina
(subendocardial ischemia) Relieved by
rest or nitroglycerin

Chest pain caused by coronary artery


Clinical features of Prinzmetal angina vasospasm. ST segment elevation.
Relieved by nitroglycerin

Non-occlusive thrombus triggers


Clinical features of unstable angina release of TXA2 (vasoconstrictor).
Occurs at rest. Risk of MI
Left anterior descending artery supplies
anterior portion of left ventricle and
anterior 2/3 of interventricular septum
(produces heart blocks) (45% of MI).
Circumflex artery, branch of left
Coronary irrigation of the heart
coronary artery (15% of MI). Right
coronary artery supplies posterior and
inferior left ventricle, right ventricle, SA
node (sinus bradycardia), papillary
muscle (mitral insuficiency) (35% of MI)

Age, family history, cigarette smoke,


Risk factors for coronary artery disease hipertension, low HDL, high LDL,
diabetes

Sudden onset of acute substernal


chest pain radiated to left arm, jaw and
Clinical presentation of AMI
neck. Shortness of breath, diapgoresis,
nausea, vomiting and anxiety

CK-MB elevated by 8h, peaks 18h,


normal in 3 days. Troponin elevated by
Serum markers of miocardial infarction 6h, peaks 16h, normal in 10 days. LDH
elevted by 24h, peaks 6 days, normal
in 14 days.

18h, no change. 24h vague pallor. 1-7d


Gross changes in miocardial infarction yellow pallor. 7-28d central pallor with
red border. Months - white firm scar

4-24h coagulative necrosis. 1-3d


Microscopic changes in miocardial neutrophilic infiltrate. 3-7d
infarction macrophages. 7-28d granulation
tissue. Months - fibrotic scar
Arrhythmias (MC COD), CHF,
pericarditis, rupture (4-7 days post-
infarct). Ventricular free wall (LAD) -->
Complications of MI cardiac tamponade. Interventricular
septum (LAD) --> left to right shunt.
Papillary muscle (RCA) mitral
insufficiency

Death within 1 hour of onset of


symptoms by fatal arrhythmia. CAD
Features of sudden cardiac death
(80%), hypertrophic cardiomyopathy,
mitral valve prolapse, aortic stenosis

Left ventricle fails --> decreased


cardiac output --> RAA system and
retention of Na and H20 --> increased
venous return causes edema and
partial compensation of CO. There's
Pathophysiology of heart failure backward pulmonary congestion that
causes dyspnea and pulmonary edema
with decreased RV output that adds up
to ystemic edema. Increased
sympathetic tone and volume retention
are compensation mechanisms
Dyspnea (due to increased pulmonary
hydrostatic pressure), pillow orthopnea
Signs and symptoms of left heart
(no gravity increases venous return
failure
with pulmonary congestion), rales, S3
gallop (volume overloaded ventricle)

Pulmonary edema, excessive RAA


Complications of left heart failure leads to secondary
hyperaldosteronism, cardiogenic shock

Due to decreased contractility after


Features and treatment of systolic left infarction. EF<0.4. Rx. Inotropics
heart failure (digitalis), decrease afterload with
vasodilators (ACE inhibitor)
Due to decreased compliance of left
ventricle (increases left atrial pressure
and pulmonary congestion). EF>0.4.
Features and treatment of diastolic left Due to left ventricular hypertrophy,
heart failure restrictive cardiomyopathy. Rx.:
increase preload by decreasing heart
rate (calcium channel blockers and B-
blockers)

Left heart failure (MCC), cor pulmonale


Causes of right heart failure
(primary pulmonary hypertension)

Jugular venous distension, nutmeg


liver hepatomegaly, dependant pitting
Clinical features of right heart failure
edema, ascites, pleural effusions,
tricuspid insuficiency

Chronic rheumatic fever is MCC. Mid-


diastolic murmur. Dyspnea and
hemoptisis (pulmonary congestion),
atrial fibrillation (left atrial dilation),
Causes and features of mitral stenosis
dysphagia for solids (enlarged left
atrium compresses esophagus),
hoarseness (irritation of recurrent
laryngeal nerve)
Valve leaflets undergo mysomatous
degeneration. Associated with lethal
ventricular arrhythmias in Marfan. Mid-
Causes and features of mitral prolapse systolic click. Infectious endocarditis
and rupture of chordae tendinae are
complications Rx.: CCA, b-blockers
and negative inotropic agents

Caused by mitral prolapse, left heart


Causes and features of mitral failure, infective endocarditis, RCA
insuficiency thrombosis (papillary muscle). Systolic
murmur, S3 heart sound.
MCC is calcified congenital bicuspid
valve, rheumatic fever, old age.
Decreased stroke volume and cardiac
output, increased afterload. Left
Causes and features of aortic stenosis ventricular hypertrophy. Systolic
murmur. Associated with angina (less
coronary filling), syncope (reduced
cardiac output) and microangiopathic
hemolytic anemia with schistocytes
MCC is essential hypertension,
infective endocarditis, syphilitic and
Causes and features of aortic
aortic aneurysms. Left ventricular
insuficiency
hypertrophy, increased preload.
Diastolic murmur, bounding pulse.

Antibodies against streptococal M


protein cross react with heart valves
Pathophysiology of rheumatic fever
producing fibrosis/stenosis, as well as
systemic features

Migratory polyarthritis, pancarditis,


Jones major criteria of rheumatic fever subcutaneous nodules, erythema
marginatum, sydenhan chrorea

Aschoff body. Fibrinoid necrosis


Pathognomonic lesion of rheumatic surrounded by macrophages
heart fever (Anitschkow cells), lymphocytes and
plasma cells

Strep viridans colonizes damaged


valves. "FROM JANE". Fever, Roth
spots on retina, Osler nodes (painful
Clinical features of subacute subcutaneous nodules on fingers and
endocarditis toes), murmur, Janeway lesions
(painless red lesions on palms and
soles), anemia, nailbed hemorrhage,
septic emboli
Associated with Turner syndrome.
Narrowing of aorta proximal to ductus
arteriosus. Ususally associated with
Preductal coarctation of the aorta PDA that supplies oxygenated blood to
distal aorta. Presents in newborn with
CHF, weak pulses and cyanosis of
lower extremities.
Narrowing of aorta distal to ductus
arteriosus. Hypertension in upper
extremities and hypotension in lower
Postductal coarctation of the aorta extremities. Can produce aortic
insuficiency, berry aneurysms and
secondat hypertension due to
increased RAA (low renal flow)

Early cyanosis due to blood shunt past


the lungs. Tetralogy of Fallot,
Right to left shunts
transposition of great vessels, truncus
arteriosus, tricuspid atresia.

Late cyanosis due to Eisenmenger


Left to right shunts
syndrome. VSD, ASD, PDA

Right side of the heart hypertrophies


due to a septal defect or PDA and
Eisenmenger syndrome
shunt reverses from left-right to right-
left producing cyanosis

Overriding aorta, pulmonic stenosis,


right ventricular hypertrophy, VSD.
Tetralogy of Fallot Cyanosis depends on degree of
pulmonic stenosis. PDA or ASD are
cardioprotective.

Inversion of aorta and pulmonary


arteries. Infants of diabetic mothers.
Transposition of the great vessels
Must have ASD, VSD or PDA to
survive.
Common pulmonary artery and aortic
trunk. Massive blood flow to the lungs
Truncus arteriosus
causes pulmonary hypertension. Early
cyanosis and CHF.

Communication between ventricles.


Large defect leads to pulmonary
VSD
hypertension and Eisenmenger
syndrome. Systolic murmur.

Communication between atriums.


ASD
Associated with fetal alcohol syndrome.

Communication between aorta and


pulmonary artery. Associated with
congenital rubella. During pregnancy
PDA
PDA is kept by PGE2. Close with
indomethacin. Machinery murmur.
Eisenmenger syndrome.

Idiopathic, postpartum, alcohol,


Coxackie B infections, doxorubicin and
Dilated cardiomyopathy
cocaine. Presents as CHF with
decreased ejection fraction

Cause of death in young athletes.


Autosomal dominant. Asymetrical
hypertrophy in ventricular septum.
Hypertrophic cardiomyopathy
Decreased compliance and stroke
volume. Rx.: increase preload with beta
blockers (decrease HR)
Right sided endocardial and valvular
fibrosis secondary to serotonin in
patients with carcinoid metastasis to
Carcinoid heart disease
liver. Skin flushing, diarrhea, crmaping,
bronchospasm, wheezing,
telangiectasia
1. sustained pressure increases wall
stress
List 3 causes of Right and Left
2. contraction against an increased
ventricular hypertrophy
resistance (afterload)

3. volume overload (increased preload)


Sustanined pressure can increase the
causes duplication of the sarcomeres,
wall stress of hte right and left
duplicates causes parallel thick
ventricles causing hypertrophy. What
muscles and duplication in series
affects does this have on the
lengthens muscle
sarcomeres

T/F contraction against an increase


resistance (after-load) produces true- new sarcomeres duplicate in
concentric thickening of the ventricular parallel to the long axes of the cells
wall

1. essential hypertension (most


list the 2 common causes of left common)
ventricular hypertrophy
2. aortic stenosis

1. pulmonary hypertension
list the 2 common causes of right
ventricular hypertrophy
2. pulmonary artery stenosis

T/ F volume overload (increased False- volume overload (increased


preload) causes constriction and preload) causes DILATION and
hypotrophy of the ventricular wall HYPERtrophy of the ventricular wall

1. mitral valve or aortic valve


regugitation
list 2 causes of eccentric hypertrophy
2. left to right shunting of blood
of left ventricles
(ventricular septal defect) which causes
more blood to return to the left side of
he heart
what causes eccentric hypertrophy of tricuspid valve of pulmonary valve
the right ventricle regurgitation

1. left or right sideded heart failure

2. angina (mostly LVH)


list 3 consequences of ventricular
hypertrophy 3. S4 heart sound ( correlated with the
atrial contraction in late diastole and by
blood entering a noncomplaint
ventricle)
Definition:

the heart fails when it is unable to eject congestive heart failure


blood delivered to it by the venous
system
1. left sided heart failure- most
common

2. right sides heart failure


list the types of CHF
3. biventricular heart failure

4. high output heart failure


1. left side of the heart can not eject
blood into the aorta

why is it said that left sided heart failure 2. increase in let ventricular end-
is a forward failure diastolic volume and pressure

3. backup of blood into the lugs causes


pulmonary edema
1. decreased ventricular contraction
(systolic dysfunction) causes ischemia
and myocardial fibrosis, myocarditis,
in LHF, the decrease in ventricular cardiomyopathy
contraction, noncompliance of the
ventricles and increased workload 2. noncompliant ventricles (diastolic
causes what effects dysfunction) restricts filling of the
ventricle and causes concentric LVH
and infiltration of muscle with amyloid,
iron or glycogen
1. systolic dysfunction- low ejection
fraction (<40%)

2. diastolic dysfunction-hight EF and an


S4 gallop due to increased resistance
what is the difference between systolic
to filling in late diastole so there is an
dysfunction and diastolic dysfunction
increase in left atrial pressure (there
could be a normal EF)

EF=stroke volume/ LV end-diastolic


volume , normal 55%-80%
1. lungs are congested and exude a
frothy pink transudate (edema)
what is the gross and microscopic
findings of LHF
2. alveolar macrophages contain
hemosiderin ( heart failure cells)
left sided heart failure
1. dyspnea- can not inspire
2. pulmonary edema-bibasilar
inspiratory crackles
3.*** Left side S3 heart sound- first
finding in LHF
4. mitral valve regurgitation
what are the differences on clinical 5. paroxysmal noctural dyspnea
exam of pt with LHF vs RHF
right sided heart failure
1. prominent jugular veins
2. right sides S3 sound due to volume
overload
3. tricuspid valve regurgitation
4. painful hepatomegaly
5. dependent pitting edema and ascites
the pulmonary edema in LHF will show congestion in upper lobes and alveolar
what finding on chest radiograph infiltrates

occurs in early diastole- intesity of


sound increases with expiration
why is a left sided S3 heart sound
caused by blood entering a volume
heard in LHF and when is this sound
overloaded left ventricle
heard on exam
*** this S3 heart sound is the first
cardiac finding in LHF
mitral valve regurgitates blood bc it is
caused by stretching of the valve ring
which causes a problem closing the
mitral valve during systole.
in LHF, the mitral valve regurgitates
blood, why and where is the murmur
blood entering the Left Atrium during
best heart
systole produces a pansystolic murmur
that increases in intensity on expiration

murmur best heard at the apex


choking sensation at night due to
increased venous return to the failed
left side of the heart

blood backs up in the lungs producing


what is paroxysmal noctural dyspnea
pulmonary edema
and why does it occur in LHF
relieved by standing on placing pillow
under head (pillow orthopnea) to
increase the effects of gravity on
reducing venous return to the heart
right side of the heart cannot pump
blood from the venous system to the
lungs
why is right sided heart failure
considered a backward failure
blood accumulated in the venous
system which leads to increase in
venous hydrostatic pressure
1. decreased contraction- right
ventricular infarction

2. noncompliant right ventricle - RVH


RHF can be caused by what 4
pathogenesis
3. increase afterload- LHF

4. increase preload- tricuspid valve


regurgitation

in Right sided heart failure, what


increase in the venous hydrostatic
causes the prominence of the jugular
pressure
veins

due to volume overload in the ventricle


in right sided heart failure what causes
which increases in intensity with
the Right S3 heart sound
inspiration

caused by stretching of the valve ring


which causes problem closing the
tricuspid valve during systole.
In right sided heart failure what causes
blood entering the right atrium during
the tricuspid valve regurgitation and
systole produces a pansystolic murmur
where is it best heard
that increases with inspiration

murmurs best heard on left parasternal


border

why is there painful hepatomegaly in passive liver congestion due to back up


RHF of venous blood in the central veins

dependent pitting edema and ascites


what kind of edema is seen in RHF and
caused by the increase in venous
why
hydrostatic pressure
Defintion:

Form of heart failure in which cardiac high output heart failure


output is increased compared with
values for the normal resting state
1. increase in stroke volume-
hyperthyroidism

2. decrease blood viscosity- severe


anemia

3. vasodilation of peripheral resistance


high output heart failure can be caused arterioles- thiamine deficiency, early
by what 4 condition phase of endotoxic shock

4. arteriovenous fistulas-bc AV
communications bypass the
mircocirculation which increases
venous return to the heart- trauma from
knife wound or surgical shunt from
hemodialysis
Definition:

Imbalance between myocardial O2 ischemic heart disease


demand and supply from the coronary
arteries

when doe coronary vessels fill diastole

1. Tachycardia (>180bpm) decreases


filling time, leading to ischemia.
what is the effect of tachycardia on the
coronary artery blood flow
2. decreases diastole which decreases
filling of coronary arteries
which coronary artery accounts for
about 50% of the coronary artery left anterior descending coronary artery
thomboses

1. Anterior portion of the left ventricle


the LAD supplies which portions of the
heart 2. Anterior two thirds of the
interventricular septum

1. Posteroinferior part of the left


ventricle

2. Posterior one third of the


interventricular septum
the right coronary artery supplies which
3. Right ventricle
portions of the heart
4. Posteromedial papillary muscle in
left ventricle

5. Both atrioventricular and sinoatrial


nodes

which coronary artery Supplies the


lateral wall of the left ventricle and
left cicumflex coronary artery
Accounts for 15% to 20% of coronary
artery thromboses

1. Angina pectoris (most common type)

2. Chronic ischemic heart disease


what are the 4 types of ischemic heart
disease
3. Sudden cardiac death

4. Myocardial infarction
1. Age- Men 45 years old and up,
women 55 years old and up

2. Family history of premature coronary


artery disease or stroke

3. Lipid abnormalities- Low-density


what risk factors are associated with
lipoprotein above 160 mg/dL or High-
ischemic heart disease
density lipoprotein below 35 mg/dL

4 Smoking tobacco,

5. hypertension

6. diabetes mellitus
chest pain or discomfort due to
coronary heart disease.

Angina is a symptom myocardial


ischemia. It occurs when the heart
muscle (myocardium) doesn't get as
what is angina pectoris-
much blood (hence as much oxygen)
as it needs. This usually happens
because one or more of the heart's
arteries (coronary blood vessels that
supply blood to the heart muscle) is
narrowed or blocked.
Clinical findings:

Exercise-induced substernal chest pain


lasting 30 seconds to 30 minutes

Relieved by resting or nitroglycerin


stable angina
Stress test shows ST-segment
depression.

a. stable angina
b. Prinzmetal's angina
c. unstable angina
d. myocardial infarction
1. Atherosclerotic coronary artery
disease (most common)
list 3 causes of stable angina
2. Aortic stenosis with concentric LVH

3. Hypertrophic cardiomyopathy
Pathogenesis
Subendocardial ischemia due to
decreased coronary artery blood flow
a. stable angina
a. stable angina
b. Prinzmetal's angina
c. unstable angina
d. myocardial infarction
Pathogenesis:

Intermittent coronary artery vasospasm


at rest

Vasoconstriction due to platelet


thromboxane A2 or decrease in b. Prinzmetal's angina
endothelin

a. stable angina
b. Prinzmetal's angina
c. unstable angina
d. myocardial infarction
Clinical findings:

Stress test shows ST-segment


elevation (transmural ischemia).

Responds to nitroglycerin and calcium-


b. Prinzmetal's angina
channel blocker (vasodilator)

a. stable angina
b. Prinzmetal's angina
c. unstable angina
d. myocardial infarction
Pathogenesis:
Severe, fixed, multivessel
atherosclerotic disease and
Disrupted plaques with or without
platelet nonocclusive thrombi

Clinical findings
Frequent bouts of chest pain at rest or c. unstable angina
with minimal exertion and May
progress to acute myocardial infarction

a. stable angina
b. Prinzmetal's angina
c. unstable angina
d. myocardial infarction
1. percutaneous transluminal coronary
angioplasty (PTCA) and stenting-
Balloon angioplasty dilates and
ruptures the atheromatous plaque
(Problem with restenosis)

2. Intracoronary stents- Decrease the


rate of restenosis. Most common early
complication is a localized dissection
with thrombosis
list 3 revascularization procedures for
angina pectoris
3. Coronary artery bypass graft
(CABG)- Used for multivessel coronary
artery atherosclerosis

a. Internal mammary artery graft- Best


graft patency after 10 years

b. Saphenous veins- "Arterialization" of


the vessels, fibrosis, and occlusion
common after 10 years
Definition:

Progressive CHF resulting from long-


term ischemic damage to myocardial
Chronic ischemic heart disease
tissue

Replacement of myocardial tissue with


noncontractile scar tissue
Unexpected death within 1 hour after
onset of symptoms

Pathogenesis:
Severe atherosclerotic coronary artery
what is the cause of death in a sudden
disease
cardiac death
Disrupted fibrous plaques
Absence of occlusive vessel thrombus
(>80% of cases)

Cause of death is ventricular fibrillation.


Sequence:

1. Sudden disruption of an
atheromatous plaque
describe the common pathogenesis of
a myocardial infarction 2. Exposed subendothelial collagen or
thrombogenic necrotic material

3. Platelet adhesion and eventual


formation of a platelet thrombus
1. Vasculitis (e.g., polyarteritis nodosa,
Kawasaki disease)

2. Cocaine use
list 4 less common causes of an MI
3. Embolization of plaque material

4. Thrombosis syndromes (e.g.,


antithrombin III deficiency,
polycythemia)

How does acute RF develop? after group A streptococcal pharyngitis

immunemeditated
how do immunitis develop against
type II hypersensitivity reaction
acute RF?
cell-meditated immunity type IV
What is the mos common initial migratory polyartritis most common
presentation of acute RF? initial presentation

what is the most common cause of myocarditis is the most common type
death death in acute RF? of death

What happens with RF in an acute mitral regurgitation in acute attack


attack? in chronic disease? mitral stenosis in chronic disease

How do you diagnose acute RF? with Jones criteria

1) Carditis
2) Arthritis
What are the major criteria in
3) Chorea
diagnosing RF?
4) Erythema marginatum subcutaneous
nodules

in acute RF what happens to ASO and ASO


DNase B titers? DNase B titers

Acute RF: MV most often involved


Which valve causes acute RF?
followed by AV
What is the most common cause of MV
recurrent RF
stenosis?

Atrial fibrillation
Mitral valve stenosis effects? Pulmonary venous hypertension
RHF

opening snap followed by an early to


What is the sound of MV stenosis?
mid-diastolic rumble

What is mitral regurgitation most


mitral valve prolapse
commonly caused by?

pansystolic murmur,
what is the sound of MV regurgitation? S3/S4 heart sounds
no intensity with deep held inspiration

MV prolapse is associated with which


Marfan and Ehlers-Danlos syndromes
syndromes?

-myxomatous degeneration
how does MVP occur?
-excess dermatan sulfate
heart sound of MVP? systolic click followed by murmur

What happens to the MVP sound if preload alters click and murmur
preload? relatioinship to S1/S2

Treatment of Symptomatic MVP? -blockers

what age group is Calcific Aortic Valve


patients >60 years
(AV) stenosis common in?

what sound does Aortic Valve (AV)


ejection murmur
stenosis produce?
intensity with preload
what happens with preload?
intensity with preload
preload?

Aortic Valve stenosis is the most


syncope
common valvular lesion
angina
what 2 things happen with exercise?

AV stenosis: microangiopathic
what kind of anemia is seen with AV
hemolytic anemia with schistocytes,
(Aortic Valve) stenosis?
hemoglobinuria
what is the most common cause of
isolated AV root dilation
aortic regurgitation

What happens to pulse pressure in


pulse pressure
Aortic valve regurgitation?

early diastolic murmur; bounding


What sound does Aortic valve
pulses; S3, S4 no intensity of
regurgitation make?
inspiration

AV regurgitation causes ____ because


hyperdynamic circulation
of a widened pulse pressure?

an Austin Flint murmur in AV


regurgitation indicates what should be sign for AV replacement
done?

Tricuspid Valve (TV) regurgitation


adults
happens in which age group?

TV (Tricuspid Valve) regurgitation can infective endocarditis


be caused by what? carcinoid heart disease
what sound does Tricuspid Valve panystolic murmur; S3/S4; intensity
regurgitation make? with deep held inspiration

Pulmonary valve (PV) regurgitation is


pulmonary hypertension
caused by what?

What 2 conditions does Carcinoid heart PV (Pulmonary valve) Stenosis


disease cause? TV (Tricuspid Valve) regurgitation

Infective endocarditis (IE) is most


commonly caused by which microbial streptococcus viridans
pathogen?

IE (Infective endocarditis ) is most


commonly caused by which microbial
staphylococcus aureus
pathogen in cases of IVDA
(intravenous drug abuse)?

What is the most common pathogen


that produces nosocomial and staphylococcus epidermidis
prosthetic valve infective endocarditis?

What is the most common pathogen


that produces IE in ulcerative streptococcus bovis
colitis/colorectal cancer?
What causes Tricuspid Valve
Tricuspid Valve regurgitation in IVDA is
regurgitation in IVDA (intravenous drug
due to infective endocarditis
abuse)?

Most consistent sign of Infective


Fever
endocarditis (IE) ?

microembolization
signs of IE?
immunocomplex vasculitis

in the majority of cases of Infective


Endocarditis what do the blood cultures positive in 80% of IE cases
test?

Libman-Sacks endocarditis is systemic lupus erythematosus(SLE)


associated with what conditions? (2) MV(mitral valve) regurgitation involved

marantic endocarditis is associated


sterile vegetation
with what type of vegetation? and
paraneoplastic syndrome
which syndrome?

most common cause of myocarditis


Coxsackievirus
and pericarditis?
what disease is the most common
cause of myocarditis leading to CHF in Chagas' disease
Central/South America?

doxorubicin
What drugs can cause myocarditis? daunorubicin
cocaine

in myocarditis what happens to the CK-


CK-MB
MB?
troponins I and T
troponins I and T?

coxsackievirus is the most common


Pericarditis
cause of what disease?

pain is relieved by
what releives pain in pericarditis? (2) -precordial rub
-lean forward

young woman with pericarditis and


---
effusion most likely has SLE

percardial effusion on inspiration the


neck vein has distention
systolic blood pressure
what happens to systolic blood
pressure?
incomplete filling of chambers
constrictive pericarditis = ?
pericardial knock

1. dilated
what are the 3 types of
2. hypertrophic
cardiomyopathy?
3. restrictive

What is the most common type of


dilated cardiomyopathy
cardiomyopathy?

cause of dilated cardiomyopathy? mycarditis is the most common cause

what drugs cause dilated doxorubicin


cardiomyopathy? daunorubicin

what happens to the heart in dilated


global enlargement of the heart
cardiomyopathy?

hypertrophic cardiomyopathy (HCM) is HCM (hypertrophic cardiomyopathy)


the most common cause of sudden most common cause of death in young
death in which age group of people? individuals
where is the obstruction located in
below the aortic valve
hypertrophic cardiomyopathy (HCM)?

What is the sound of HCM preload changes on murmur intensity


(hypertrophic cardiomyopathy)? opposite of those for AV stenosis

what is the cause of sudden death in sudden death due to ventricular


hypertrophic cardiomyopathy? tachycardia/ fibrillation

What is the treatment of HCM


-blockers
(hypertrophic cardiomyopathy)?

restrictive cardiomyopathy which is the


least common cardiomyopathy has low voltatge ECG
what type of voltatge ECG

what happens to ventricular


compliance in Restrictive ventricular compliance
cardiomyopathy ?

which is more common in heart tumors


metastasis > primary tumors
metastasis or primary tumors?
cardiac myxoma most common in
in left atrium
which atrium?

myxomas occur which age group? myxomas occur in adults


rhabdomyomas occur at what age? rhabdomyomas occur in children

what effect does wall stress have on wall stress increases gene-controlled
sarcomere duplication?" sarcomere duplication

resistance ventricle contracts against to


define afterload = ?
eject blood in systole

what is the ventricular hypertrophy in increased afterload causes concentric


relation to afterload? hypertrophy

equivalent to LVEDV (left ventricular


preload = ?
end diastolic volume)

what is ventricular hypertrophy in increased preload causes eccentric


relation to preload? hypertrophy
heart failure
what are the consequences of
S[subscript]4
hypertrophy? (3)
angina (LVH)

what is S[subscript]4? blood entering noncompliant ventricle

left side failure= forward failure ->


what does left side failure lead to?
pulmonary endema

what is the most common type of LHF


systolic dysfunction
(Left Heart Failure)?

what is the most common type of


Diastolic dysfunction
hypertension?

ventricular contraction
Systolic dysfunction = ?
(Ejection faction) EF

resistance to filling the ventricle


Diastolic dysfunction - what happens?
normal EF (ejection faction)
What happens with dyspnea? cannot take full inspiration

pulmonary edema = ? hydrostatic pressure > oncotic pressure

Kerley's lines indicates what? septal edema

alveolar macrophages with


Heart failure cells = ?
hemosiderin

S [underscore]3 heart sound = ? first cardiac sign of LHF

PND (Paroxysmal nocturnal dyspnea)/ venous return to right


orthopnea = ? side of the heart at night

What is brain natriuretic peptide (BNP) BNP (brain natriuretic peptide) is useful
useful for? for confirming/ excluding LHF
RHF = backward failure -> increase in
RHF = ?
venous hydrostatic pressure

what is the most common cause of


RHF
LHF?

what happens to venous hydrostatic


venous hydrostatic pressure
pressure in RHF?

neck vein distention


hepatomegaly
Clinical findings of RHF?
dependent pitting edema
ascites

ACE (angiotensin-converting enzyme) afterload


inhibitors causes? preload

myocardial O2 consumption
-Blockers cause?
heart rate

SV (stroke volume)
High output failure causes? TPR (total peripheral resistance)
arteriovenous fistula
decreases diastole and filling of
Tachycardia does what?
coronary arteries

What is the most common site of LAD (Left anterior descending)


coronary artery thombosis? coronary artrery

Angina pectoris is the most common


how is Angina pectoris related to
manifestation of coronary artery
coronary artery disease?
disease?

what is the most important risk factor in age is the most important risk factor in
Angina pectoris Angina pectoris

which sex Angina pectoris most


males> females
prevalent in?

what is the most common type of


stable angina
angina

it is exercise induced substernal chest


Define stable angina:
pain
subendocardial ischemia with ST-
Stable angina is caused by what?
segment depression

vasospasm with transmural ischemia


Prinzmetal's angina = ?
and ST-segment elevation

angina at rest; multivessel disease,


Unstable angina = ?
disrupted plaques

calcium channel blockers vasodilate


What causes Prinzmetal's angina?
coronary arteries

replacement of muscle by fibrous


What is chronic ischemic heart disease
tissue

unexpected death within 1 hour after


Define sudden cardiac death
symptoms

what is usually not present at sudden


coronary artery thrombosis
cardiac arrest?
MVP (mitral valve prolapse) sudden arrhythmias from mitral regurgitation or
death is caused by? CHF (congestive heart failure)

What is the most common cause of


AMI (Acute myocardial infarction)
death in the US?

what is the sequence of AMI (Acute Rupture of disrupted plaque -> platelet
myocardial infarction)? -> AMI (Acute myocardial infarction)

What does AMI (Acute myocardial


AMI with normal coronary arteries
infarction) by cocaine look like?

what is the significance of Q wave in Q wave type transmural


AMI non-Q wave type subendocardial

what is the significance of reperfusion


short/long term survival
in AMI

reperfusion histologically alters the


damaged cells
Contraction band necrosis = ?
hypercontraction myofibrils due to
Ca2+
How long does the AMI coagulation AMI : coagulation necrosis within 24
take hours

3-7 days, at which time there is danger


during AMI when is the heart softest?
of rupture

retrosternal pain, radiation to left arm/


Clinical findings of AMI?
shoulder, diaphoresis

if Q wave AMI is present? early mortality rate

Non-Q wave AMI is present? risk for SCD (sudden cardiac death)

Ventricular fibrillation relates to AMI Ventricular fibrillation is the most


how? common cause of death in acute MI

When is Myocardial rupture common? 3-7 days


What happens when posteromdial RCA thrombosis;
papillary muscle ruptures? mitral regurgitation

Mural thrombus is not a danger to


F, it is a danger to embolization
embolization T/F

early (acute inflammation)


Fibrinous pericarditis?
and late complication (autoimmune)

CHF most common cause of death? ventricular aneurysm

hypotension
RV AMI Clinical findings? RHF
preserved LV function

reappearance of CK-MB (creatin


Reinfarction?
kinase isoenzyme) after 3 days

cTnI and cTnT cannot diagnose


cTnI and cTnT in relation to reinfarction
reinfarction
cTnI and cTnT is the gold standard for
cTnI and cTnT used for AMI
AMI

inverted T waves
What are the ECG findings in AMI? elevated ST segment
Q waves

chorionic villus is the primary site for


Chorionic villus is the primary site for?
O2 exchange

Umbilical vein relating to CHD


is the highest Po2 in fetal circulation
(congenital heart disease)?

foramen ovale and ductus arteriosus


Fetal circulation = ?
are patent

Single umbilical artery = ? risk congenital abnormalities

ASD (atrial septal defect) is fixed


ASD (atrial septal defect) heart sound
splitting of S2;
=?
most common adult CHD
PDA (patent ductus arteriosus) closed with indomethacin or surgery
treatment and sound? machinery murmur

What is the most common cyanotic


Tetralogy of Fallot
CHD?

In Tetralogy of Fallot what does the degree of PV stenosis correlates to


degree of PV stenosis correlate to? absence of cyanosis

ASD (atrial septal defect) - steps up


Which cardioprotective shunts help Sao2 in right atrium
CHD? PDA (patent ductus arteriosus) - shunts
blood from aorta -> pulmonary artery

squatting systemic vascular


what can Tet spells do in CHD?
resistance; PaO2

aorta emties RV, pulmonary artery


transposition = ?
empties LV, atria normal

What is infantile coarctation associated


Turner's syndrome
with?
disparity between upper/lower
What happens in adult coarctation?
extremity blood pressure> 10 mm Hg

How does hypertension develop in due to activation RAA (renin-


adult coarctation? angiotensin-aldosterone) system

AIA(anterior intercostal arteries)-


PIA(posterior intercostal arteries) to
aorta
Coarctation collaterals: where do they
develop?
SEA(superior epigastric artery)-
IEA(inferior epigastric artery) to
external iliac artery

You might also like